Find the values of a>0 for which the improper integral $int_{0}^{infty}frac{sin x}{x^{a}} $ converges ....

Processor speed limited at 0.4 Ghz

Notepad++ delete until colon for every line with replace all

How can a day be of 24 hours?

How to prevent "they're falling in love" trope

How does a dynamic QR code work?

Finitely generated matrix groups whose eigenvalues are all algebraic

What Exploit Are These User Agents Trying to Use?

Where would I need my direct neural interface to be implanted?

how do we prove that a sum of two periods is still a period?

How seriously should I take size and weight limits of hand luggage?

Why do I get negative height?

Is it a bad idea to plug the other end of ESD strap to wall ground?

Knowledge-based authentication using Domain-driven Design in C#

Rotate ASCII Art by 45 Degrees

ssTTsSTtRrriinInnnnNNNIiinngg

Machine learning testing data

Fair gambler's ruin problem intuition

Was the old ablative pronoun "med" or "mēd"?

Placement of More Information/Help Icon button for Radio Buttons

What are the G forces leaving Earth orbit?

What exactly is ineptocracy?

Forgetting the musical notes while performing in concert

What historical events would have to change in order to make 19th century "steampunk" technology possible?

What is an equivalently powerful replacement spell for the Yuan-Ti's Suggestion spell?



Find the values of a>0 for which the improper integral $int_{0}^{infty}frac{sin x}{x^{a}} $ converges . [duplicate]


Convergence $I=int_0^infty frac{sin x}{x^s}dx$Improper Integral:$int_{0}^{+infty}frac{sin x}{x+sin x}dx$Improper integral $sin(x)/x $ converges absolutely, conditionaly or diverges?For which values is this improper integral convergent?Does the improper integral $int_{0}^{infty}sin(x^2);mathrm dx$ converge?Improper integral: $int_0^infty frac{sin^4x}{x^2}dx$Determining for which positive values of $p$ the improper integral $int_1^{infty}frac{dx}{e^{px} ln{x}}$ convergesImproper integral - checking convergence of $int_{1}^{infty} x^2 sin(x^4) dx$Can't solve Improper Integral $int_{0}^{infty} frac{sqrt{x}sin(x)}{1+x^2} dx$Find all the values of $alpha$ and $beta$ for which the following integral convergesFor what values of ''$a$'' , does this Improper Integral converges?













-4












$begingroup$



This question already has an answer here:




  • Convergence $I=int_0^infty frac{sin x}{x^s}dx$ [duplicate]

    5 answers




Find the values of a>0 for which the improper integral $int_{0}^{infty}frac{sin x}{x^{a}} $ converges .
Do I have to expand integrand using series expansion??










share|cite|improve this question











$endgroup$



marked as duplicate by mrtaurho, uniquesolution, John Omielan, StubbornAtom, Crostul Mar 18 at 20:33


This question has been asked before and already has an answer. If those answers do not fully address your question, please ask a new question.


















  • $begingroup$
    Not necessarily.
    $endgroup$
    – uniquesolution
    Mar 18 at 13:50










  • $begingroup$
    Tell me another way !!
    $endgroup$
    – sejy
    Mar 18 at 13:55






  • 2




    $begingroup$
    @sejy you shouldn't use this tone against people that you want help from for free
    $endgroup$
    – Henry Lee
    Mar 18 at 14:07










  • $begingroup$
    math.stackexchange.com/questions/793595/…
    $endgroup$
    – StubbornAtom
    Mar 18 at 14:29
















-4












$begingroup$



This question already has an answer here:




  • Convergence $I=int_0^infty frac{sin x}{x^s}dx$ [duplicate]

    5 answers




Find the values of a>0 for which the improper integral $int_{0}^{infty}frac{sin x}{x^{a}} $ converges .
Do I have to expand integrand using series expansion??










share|cite|improve this question











$endgroup$



marked as duplicate by mrtaurho, uniquesolution, John Omielan, StubbornAtom, Crostul Mar 18 at 20:33


This question has been asked before and already has an answer. If those answers do not fully address your question, please ask a new question.


















  • $begingroup$
    Not necessarily.
    $endgroup$
    – uniquesolution
    Mar 18 at 13:50










  • $begingroup$
    Tell me another way !!
    $endgroup$
    – sejy
    Mar 18 at 13:55






  • 2




    $begingroup$
    @sejy you shouldn't use this tone against people that you want help from for free
    $endgroup$
    – Henry Lee
    Mar 18 at 14:07










  • $begingroup$
    math.stackexchange.com/questions/793595/…
    $endgroup$
    – StubbornAtom
    Mar 18 at 14:29














-4












-4








-4





$begingroup$



This question already has an answer here:




  • Convergence $I=int_0^infty frac{sin x}{x^s}dx$ [duplicate]

    5 answers




Find the values of a>0 for which the improper integral $int_{0}^{infty}frac{sin x}{x^{a}} $ converges .
Do I have to expand integrand using series expansion??










share|cite|improve this question











$endgroup$





This question already has an answer here:




  • Convergence $I=int_0^infty frac{sin x}{x^s}dx$ [duplicate]

    5 answers




Find the values of a>0 for which the improper integral $int_{0}^{infty}frac{sin x}{x^{a}} $ converges .
Do I have to expand integrand using series expansion??





This question already has an answer here:




  • Convergence $I=int_0^infty frac{sin x}{x^s}dx$ [duplicate]

    5 answers








improper-integrals






share|cite|improve this question















share|cite|improve this question













share|cite|improve this question




share|cite|improve this question








edited Mar 18 at 13:54









Bernard

124k741118




124k741118










asked Mar 18 at 13:47









sejysejy

1589




1589




marked as duplicate by mrtaurho, uniquesolution, John Omielan, StubbornAtom, Crostul Mar 18 at 20:33


This question has been asked before and already has an answer. If those answers do not fully address your question, please ask a new question.









marked as duplicate by mrtaurho, uniquesolution, John Omielan, StubbornAtom, Crostul Mar 18 at 20:33


This question has been asked before and already has an answer. If those answers do not fully address your question, please ask a new question.














  • $begingroup$
    Not necessarily.
    $endgroup$
    – uniquesolution
    Mar 18 at 13:50










  • $begingroup$
    Tell me another way !!
    $endgroup$
    – sejy
    Mar 18 at 13:55






  • 2




    $begingroup$
    @sejy you shouldn't use this tone against people that you want help from for free
    $endgroup$
    – Henry Lee
    Mar 18 at 14:07










  • $begingroup$
    math.stackexchange.com/questions/793595/…
    $endgroup$
    – StubbornAtom
    Mar 18 at 14:29


















  • $begingroup$
    Not necessarily.
    $endgroup$
    – uniquesolution
    Mar 18 at 13:50










  • $begingroup$
    Tell me another way !!
    $endgroup$
    – sejy
    Mar 18 at 13:55






  • 2




    $begingroup$
    @sejy you shouldn't use this tone against people that you want help from for free
    $endgroup$
    – Henry Lee
    Mar 18 at 14:07










  • $begingroup$
    math.stackexchange.com/questions/793595/…
    $endgroup$
    – StubbornAtom
    Mar 18 at 14:29
















$begingroup$
Not necessarily.
$endgroup$
– uniquesolution
Mar 18 at 13:50




$begingroup$
Not necessarily.
$endgroup$
– uniquesolution
Mar 18 at 13:50












$begingroup$
Tell me another way !!
$endgroup$
– sejy
Mar 18 at 13:55




$begingroup$
Tell me another way !!
$endgroup$
– sejy
Mar 18 at 13:55




2




2




$begingroup$
@sejy you shouldn't use this tone against people that you want help from for free
$endgroup$
– Henry Lee
Mar 18 at 14:07




$begingroup$
@sejy you shouldn't use this tone against people that you want help from for free
$endgroup$
– Henry Lee
Mar 18 at 14:07












$begingroup$
math.stackexchange.com/questions/793595/…
$endgroup$
– StubbornAtom
Mar 18 at 14:29




$begingroup$
math.stackexchange.com/questions/793595/…
$endgroup$
– StubbornAtom
Mar 18 at 14:29










2 Answers
2






active

oldest

votes


















2












$begingroup$

For $a ge 2$ the singularity at $0$ is non-integrable. For $1 < a < 2$ it's absolutely integrable both at $0$ and at $infty$. For $0 < a le 1$ the series $sum_{n=1}^infty int_{npi}^{(n+1)pi} frac{sin(x)}{x^a}; dx$ is an alternating series.






share|cite|improve this answer









$endgroup$





















    0












    $begingroup$

    We know that $-1lesin(x)le 1$ and so we can approximate the integral with:
    $$int_0^inftyfrac{sin(x)}{x^a}dxleint_0^inftyfrac{1}{x^a}$$
    and this integral is convergent for $a>1$ so this narrows our region down to somewhere in $0<a<1$






    share|cite|improve this answer









    $endgroup$









    • 2




      $begingroup$
      It converges also for $alpha=1$
      $endgroup$
      – HAMIDINE SOUMARE
      Mar 18 at 14:16








    • 1




      $begingroup$
      @ Henry Lee You should better consider absolute values in your inequalitiy: $| int_0^infty frac{sin(x)}{x^alpha},dx | le int_0^infty |frac{sin(x)}{x^alpha}|,dxle int_0^infty frac{1}{x^alpha},dx$
      $endgroup$
      – Dr. Wolfgang Hintze
      Mar 18 at 14:50








    • 1




      $begingroup$
      For general $alpha$ the integral defines the function $cos left(frac{pi alpha}{2}right) Gamma (1-alpha)$ by analytic continuation.
      $endgroup$
      – Dr. Wolfgang Hintze
      Mar 18 at 14:53




















    2 Answers
    2






    active

    oldest

    votes








    2 Answers
    2






    active

    oldest

    votes









    active

    oldest

    votes






    active

    oldest

    votes









    2












    $begingroup$

    For $a ge 2$ the singularity at $0$ is non-integrable. For $1 < a < 2$ it's absolutely integrable both at $0$ and at $infty$. For $0 < a le 1$ the series $sum_{n=1}^infty int_{npi}^{(n+1)pi} frac{sin(x)}{x^a}; dx$ is an alternating series.






    share|cite|improve this answer









    $endgroup$


















      2












      $begingroup$

      For $a ge 2$ the singularity at $0$ is non-integrable. For $1 < a < 2$ it's absolutely integrable both at $0$ and at $infty$. For $0 < a le 1$ the series $sum_{n=1}^infty int_{npi}^{(n+1)pi} frac{sin(x)}{x^a}; dx$ is an alternating series.






      share|cite|improve this answer









      $endgroup$
















        2












        2








        2





        $begingroup$

        For $a ge 2$ the singularity at $0$ is non-integrable. For $1 < a < 2$ it's absolutely integrable both at $0$ and at $infty$. For $0 < a le 1$ the series $sum_{n=1}^infty int_{npi}^{(n+1)pi} frac{sin(x)}{x^a}; dx$ is an alternating series.






        share|cite|improve this answer









        $endgroup$



        For $a ge 2$ the singularity at $0$ is non-integrable. For $1 < a < 2$ it's absolutely integrable both at $0$ and at $infty$. For $0 < a le 1$ the series $sum_{n=1}^infty int_{npi}^{(n+1)pi} frac{sin(x)}{x^a}; dx$ is an alternating series.







        share|cite|improve this answer












        share|cite|improve this answer



        share|cite|improve this answer










        answered Mar 18 at 14:13









        Robert IsraelRobert Israel

        330k23219473




        330k23219473























            0












            $begingroup$

            We know that $-1lesin(x)le 1$ and so we can approximate the integral with:
            $$int_0^inftyfrac{sin(x)}{x^a}dxleint_0^inftyfrac{1}{x^a}$$
            and this integral is convergent for $a>1$ so this narrows our region down to somewhere in $0<a<1$






            share|cite|improve this answer









            $endgroup$









            • 2




              $begingroup$
              It converges also for $alpha=1$
              $endgroup$
              – HAMIDINE SOUMARE
              Mar 18 at 14:16








            • 1




              $begingroup$
              @ Henry Lee You should better consider absolute values in your inequalitiy: $| int_0^infty frac{sin(x)}{x^alpha},dx | le int_0^infty |frac{sin(x)}{x^alpha}|,dxle int_0^infty frac{1}{x^alpha},dx$
              $endgroup$
              – Dr. Wolfgang Hintze
              Mar 18 at 14:50








            • 1




              $begingroup$
              For general $alpha$ the integral defines the function $cos left(frac{pi alpha}{2}right) Gamma (1-alpha)$ by analytic continuation.
              $endgroup$
              – Dr. Wolfgang Hintze
              Mar 18 at 14:53


















            0












            $begingroup$

            We know that $-1lesin(x)le 1$ and so we can approximate the integral with:
            $$int_0^inftyfrac{sin(x)}{x^a}dxleint_0^inftyfrac{1}{x^a}$$
            and this integral is convergent for $a>1$ so this narrows our region down to somewhere in $0<a<1$






            share|cite|improve this answer









            $endgroup$









            • 2




              $begingroup$
              It converges also for $alpha=1$
              $endgroup$
              – HAMIDINE SOUMARE
              Mar 18 at 14:16








            • 1




              $begingroup$
              @ Henry Lee You should better consider absolute values in your inequalitiy: $| int_0^infty frac{sin(x)}{x^alpha},dx | le int_0^infty |frac{sin(x)}{x^alpha}|,dxle int_0^infty frac{1}{x^alpha},dx$
              $endgroup$
              – Dr. Wolfgang Hintze
              Mar 18 at 14:50








            • 1




              $begingroup$
              For general $alpha$ the integral defines the function $cos left(frac{pi alpha}{2}right) Gamma (1-alpha)$ by analytic continuation.
              $endgroup$
              – Dr. Wolfgang Hintze
              Mar 18 at 14:53
















            0












            0








            0





            $begingroup$

            We know that $-1lesin(x)le 1$ and so we can approximate the integral with:
            $$int_0^inftyfrac{sin(x)}{x^a}dxleint_0^inftyfrac{1}{x^a}$$
            and this integral is convergent for $a>1$ so this narrows our region down to somewhere in $0<a<1$






            share|cite|improve this answer









            $endgroup$



            We know that $-1lesin(x)le 1$ and so we can approximate the integral with:
            $$int_0^inftyfrac{sin(x)}{x^a}dxleint_0^inftyfrac{1}{x^a}$$
            and this integral is convergent for $a>1$ so this narrows our region down to somewhere in $0<a<1$







            share|cite|improve this answer












            share|cite|improve this answer



            share|cite|improve this answer










            answered Mar 18 at 14:11









            Henry LeeHenry Lee

            2,189319




            2,189319








            • 2




              $begingroup$
              It converges also for $alpha=1$
              $endgroup$
              – HAMIDINE SOUMARE
              Mar 18 at 14:16








            • 1




              $begingroup$
              @ Henry Lee You should better consider absolute values in your inequalitiy: $| int_0^infty frac{sin(x)}{x^alpha},dx | le int_0^infty |frac{sin(x)}{x^alpha}|,dxle int_0^infty frac{1}{x^alpha},dx$
              $endgroup$
              – Dr. Wolfgang Hintze
              Mar 18 at 14:50








            • 1




              $begingroup$
              For general $alpha$ the integral defines the function $cos left(frac{pi alpha}{2}right) Gamma (1-alpha)$ by analytic continuation.
              $endgroup$
              – Dr. Wolfgang Hintze
              Mar 18 at 14:53
















            • 2




              $begingroup$
              It converges also for $alpha=1$
              $endgroup$
              – HAMIDINE SOUMARE
              Mar 18 at 14:16








            • 1




              $begingroup$
              @ Henry Lee You should better consider absolute values in your inequalitiy: $| int_0^infty frac{sin(x)}{x^alpha},dx | le int_0^infty |frac{sin(x)}{x^alpha}|,dxle int_0^infty frac{1}{x^alpha},dx$
              $endgroup$
              – Dr. Wolfgang Hintze
              Mar 18 at 14:50








            • 1




              $begingroup$
              For general $alpha$ the integral defines the function $cos left(frac{pi alpha}{2}right) Gamma (1-alpha)$ by analytic continuation.
              $endgroup$
              – Dr. Wolfgang Hintze
              Mar 18 at 14:53










            2




            2




            $begingroup$
            It converges also for $alpha=1$
            $endgroup$
            – HAMIDINE SOUMARE
            Mar 18 at 14:16






            $begingroup$
            It converges also for $alpha=1$
            $endgroup$
            – HAMIDINE SOUMARE
            Mar 18 at 14:16






            1




            1




            $begingroup$
            @ Henry Lee You should better consider absolute values in your inequalitiy: $| int_0^infty frac{sin(x)}{x^alpha},dx | le int_0^infty |frac{sin(x)}{x^alpha}|,dxle int_0^infty frac{1}{x^alpha},dx$
            $endgroup$
            – Dr. Wolfgang Hintze
            Mar 18 at 14:50






            $begingroup$
            @ Henry Lee You should better consider absolute values in your inequalitiy: $| int_0^infty frac{sin(x)}{x^alpha},dx | le int_0^infty |frac{sin(x)}{x^alpha}|,dxle int_0^infty frac{1}{x^alpha},dx$
            $endgroup$
            – Dr. Wolfgang Hintze
            Mar 18 at 14:50






            1




            1




            $begingroup$
            For general $alpha$ the integral defines the function $cos left(frac{pi alpha}{2}right) Gamma (1-alpha)$ by analytic continuation.
            $endgroup$
            – Dr. Wolfgang Hintze
            Mar 18 at 14:53






            $begingroup$
            For general $alpha$ the integral defines the function $cos left(frac{pi alpha}{2}right) Gamma (1-alpha)$ by analytic continuation.
            $endgroup$
            – Dr. Wolfgang Hintze
            Mar 18 at 14:53





            Popular posts from this blog

            Magento 2 - Add success message with knockout Planned maintenance scheduled April 23, 2019 at 23:30 UTC (7:30pm US/Eastern) Announcing the arrival of Valued Associate #679: Cesar Manara Unicorn Meta Zoo #1: Why another podcast?Success / Error message on ajax request$.widget is not a function when loading a homepage after add custom jQuery on custom themeHow can bind jQuery to current document in Magento 2 When template load by ajaxRedirect page using plugin in Magento 2Magento 2 - Update quantity and totals of cart page without page reload?Magento 2: Quote data not loaded on knockout checkoutMagento 2 : I need to change add to cart success message after adding product into cart through pluginMagento 2.2.5 How to add additional products to cart from new checkout step?Magento 2 Add error/success message with knockoutCan't validate Post Code on checkout page

            Fil:Tokke komm.svg

            Where did Arya get these scars? Unicorn Meta Zoo #1: Why another podcast? Announcing the arrival of Valued Associate #679: Cesar Manara Favourite questions and answers from the 1st quarter of 2019Why did Arya refuse to end it?Has the pronunciation of Arya Stark's name changed?Has Arya forgiven people?Why did Arya Stark lose her vision?Why can Arya still use the faces?Has the Narrow Sea become narrower?Does Arya Stark know how to make poisons outside of the House of Black and White?Why did Nymeria leave Arya?Why did Arya not kill the Lannister soldiers she encountered in the Riverlands?What is the current canonical age of Sansa, Bran and Arya Stark?